Emancipation and Other Pursuits

Freedom is not a binary thing. It can be defined, but its definition need not be all-encompassing. Freedom can include as severe restrictions and unpleasant experience as there was before, but it is only the perspective that has changed. A rather simple example of this is virtually all of Sefer Shemot, where we have an emancipation of our people from slavery, only to be constrained and shackled by a new kind of slavery, our lives once again compelled and seemingly obligated to go in a certain direction, to conduct and comport ourselves according to a set of rules and strictures that bound those of us who came after to a lifestyle and a dictum that forces our conduct to go beyond all means of rationality.  And, as has been the case for some time, true freedom, of action and of perspective, eludes our grasp.

Honestly Frum made an interesting observation on my last post. He wondered, quite rightly, why it is not possible to accept there are things we don’t know and, as he put it rather bluntly “Who made Hashem? Frankly, who cares?” I think what he said and what he means is rather seductive, and it appeals to me as much as I think it appeals to him. And I could go down that road. It would be easy, understandable even, to conceal my doubts, my conclusions, my hard-fought understanding beneath a blanket of yet more uncertainty, but this is uncertainty is said to be of a benign nature, of the wonderment of that which we don’t understand. And I simply cannot do that. Not anymore. What I want for me and if I am fortunate, my son, is to be free of being forced to hope and to pray that Hashem really is out there; that I can impute Hashem into the vast uncertainty that those of us who understand confront every day. When I daven in Shul, when I get an aliyah, I stand tall and proud before the Torah, and pronounce the brachot with all the certainty that one would expect from a Rabbi. But in my heart, a place of a deep and abiding skepticism, Honestly Frum wants me fill it with wonder, or with apathy or with concealment, finding God in the proverbial details and letting the big questions skulk away in the shadow of small answers. I just can’t do it.

Emancipation did not work for the slaves; it took one hundred years until The Civil Rights Act was passed and even today, there are difficulties with that gracious and overdue act. Emancipation did not work for the Jews in Egypt either, as discussed above. What I want, desperately, for my son, is not to have to make those types of compromises and to be free, if he so chooses, to find the spirit of God or Hashem or whomever in whatever he sees around him. Emancipation doesn’t just happen because he knows who created God; it is a freedom to not have to think about that question at all.

This entry was posted in Uncategorized. Bookmark the permalink.

113 Responses to Emancipation and Other Pursuits

  1. Jacob Stein says:

    No slavery is more brutal than the slavery of atheists to their addictions.

    Read for example Tweak

    The Heroin Diaries

    or Porn Nation

  2. amldc says:

    Thanks again for writing this blog. I wonder, with this entry, if the fatal flaw here is assuming we are ever free, God or no god.

  3. Min HaMeitzar says:

    I feel sorry for you….for the situation you find yourself in, with no one to confide in – especially, and most tellingly and most psychologically difficult, your wife. You thus (and this is apparent from what you’ve written) really have no life partner, and so you are alone – terribly alone. This factor ought to give you pause – you are, after all, highly intelligent, and the outcome is not hard to see. Sooner or later the charade you maintain will grow more and more difficult to carry forward. One day, perhaps, you will forget to close out your computer and someone – your wife? Your children? A congregant? – will find you out, and it will be over.

    In a larger sense, though, you are not the only thinking person who has been beset by challenges to faith. There are very fine minds who have gone before you – and they have not been overcome by the “thousand cuts” that you seem to have succumbed to. You are not more intelligent, to put it mildly, than Rav Soloveichik, than R’ Akiva Eiger, and so on, all of whom were quite happy to remain believers. It takes a certain arrogance to declare oneself in defiance of the sublime minds that have graced our history.

    What I think has happened (and it happened to me) is that the increasing absurdities of what passes for “orthodox” or “hareidi” Judaism today has driven people to question the faith itself. A world in which strawberries, asparagus, broccoli, ((and now corn on the cob, according to the esteemed Rabbi Vaye) can no longer be eaten is a world run amok. And it has run amok in a myriad of ways…I was speaking recently to a gentleman who is of the “gadol” variety, and he said to me: do you think I don’t question where Halacha has taken us – do you think that I don’t see where it has developed in the wrong direction?

    And this is where, I think, the way opens for disbelief. When extremists rule the playing field, when there is no room for dissent, the masses begin to question the entire system, a phenomenon which is clearly in evidence in the many comments left on sites like “Vos is Neias”, and so on. I was there, and I pulled back…I came to the realization that the religion is what you make of it. In it’s purest form, it enjoins us to help others, to make the world better, and to aspire to be better people. It tells us to make our wives and families the focal points of our lives (and not to spend more time with our ” night chavrusaos” than our wives and families). It tells us that the world is a fabulously beautiful place that we are to enjoy in all it’s dimensions. That’s the yiddishkeit of my youth, growing up in small town America, and it has disappeared.

    We lead lives today of incredible frustration and stress as Jews. Tuitions are high, and the demands placed by Jewish life on us are almost unbearable. Your place as a rabbi is to help your congregants survive…your place as a father and husband is to be a father and, especially, a husband.

    • Eli says:

      Thinking your own thoughts through honestly is not arrogance. It’s what you’re supposed to do if you have integrity. You follow what you see as the truth wherever it takes you. Rabbi J. Soloveitchik was an ultra brain wave for sure, but replacing faith in G-d with faith in the Rav will not make the questions go away. There is no reverence for G-d without reverence for truth.

      • Min HaMeitzar says:

        Eli…the problem with the statement that “you follow what you see as the truth wherever it takes you” and that this is called “integrity” is that it is entirely subjective. “you” – “your” thinking – is ultimately only based on you…and you are just not that smart. By that I mean, of course, that whatever conclusions you reach are only based on some combination of your intellect and what experience/learning you have achieved and assimilated into your being. Then there is the emotional component of yourself that further colors how make judgments and reach conclusions. I will absolutely guarantee that no conclusion that you reach will ever be more accurate than that of Rav Soloveichik’s. He simply was smarter than you , knew more than you – and I refer to the entire gamut of human knowledge , not merely “Jewish knowledge”, and had acute reasoning capabilities that were transcendent. What is true is that every person should question everything, and should use their intellect to the utmost of their capability. Yet our intellects are only that – our own – and we have to recognize – and be humbled – by that reality. It is, sadly, arrogant to think otherwise.

        We have a religion that at it’s very core was deeply satisfying – and is deeply satisfying – to astonishingly sophisticated minds. That ought to give us pause. As I noted earlier, the current “face” of the religion is troubling in it’s extremist look and practice….the proper reaction is to focus on the core message, ignore the nonsense, and do the right thing for yourself and for others.

    • shim (Shimon) says:

      I concur.
      I would just like to add a question/suggestion:
      Why don’t you completely open up to your wife? I mean you are married and I assume that you love each other right? if so then while she may not be happy with your conclusion why assume that she wouldn’t understand or something to that effect? I think you should and try to find a middle ground at least for your family’s decision for how to raise your children, because it is obvious how much you care about them

    • Sam says:

      >You are not more intelligent, to put it mildly, than Rav Soloveichik, than R’ Akiva Eiger, and so on, all of whom were quite happy to remain believers. It takes a certain arrogance to declare oneself in defiance of the sublime minds that have graced our history.

      For close to 2000 years, Aristotelian science dominated all science, and no university would teach anything else. What kind of upstarts would overturn that foundation of any reputable academy, which was accepted by so many brilliant minds? Answer: those who realized it was wrong.

      Besides which, for every brilliant Orthodox Jew you can name, there have been (statistically speaking) a greater number of brilliant people who weren’t Orthodox Jews, or who believed things that were wrong, etc.

    • G*3 says:

      > You are not more intelligent, to put it mildly, than Rav Soloveichik, than R’ Akiva Eiger, and so on, all of whom were quite happy to remain believers.

      You are not more intelligent, to put it mildly, than Sir Issac Newton, than St. Thomas Aquinas, and so on, all of whom were quite happy to remain Christians. It takes a certain arrogance to declare oneself an Orthodox Jew in defiance of the sublime minds that have graced history.

      Min HaMeitzar, I’m curious: you say in your comment that the absurdities of ever-increasing chumras may “people to question the faith itself,” but that “the religion is what you make of it” and by focusing on the good parts you can make it something beautiful.

      What does the run-away stringency or the beauty of a belief system have to do with anything? Either God is real or He isn’t. Either God gave the Torah to Moshe on Har Sinia or He didn’t. Either the rabbonim were given the power to interpret the Torah or they weren’t. If these things are true, then we are obliged to follow the psak of the rabbonim, even if they decree ever more stringent chumras. If these things are not true, then all of Yiddishkeit can be safely discarded, no matter how beautiful one may be able to make it.

      • Min HaMeitzar says:

        Ah yes…Sir Isaac Newton (whose fascination with things Jewish is well documented)…I would venture to say that this quote (from the Microsoft Encarta) is of interest:

        Newton also wrote on Judaeo-Christian prophecy, whose decipherment was essential, he thought, to the understanding of God. His book on the subject, which was reprinted well into the Victorian Age, represented lifelong study. Its message was that Christianity went astray in the 4th century AD, when the first Council of Nicaea propounded erroneous doctrines of the nature of Christ. The full extent of Newton’s unorthodoxy was recognized only in the present century: but although a critic of accepted Trinitarian dogmas and the Council of Nicaea, he possessed a deep religious sense, venerated the Bible and accepted its account of creation. In late editions of his scientific works he expressed a strong sense of God’s providential role in nature.

        I am not an expert on the subject, but would say that for thousands of years there has been an unparalleled intellectual engagement – generation after generation after generation – that has focused on mankind’s relationship to God. That has been a Jewish undertaking. There is no parallel to it in the Christian world. The finest and best in that world have not been found among the religious leadership, and indeed we know of centuries of corruption in that leadership (we, evidently, are trying to catch up in these latter days). A person like Rav Soloveichik was not merely born brilliant, not merely well educated – he was the product of centuries and centuries of intellectual development and effort. I know of no parallel elsewhere.

        So – yep, there was a Torah given at Sinai. The rabbis were absolutely NOT given any power – anymore than you or I – to interpret anything. There was an interpretive tradition that hearkened back to the very beginning…and anyone – you or I – that was willing to expend the effort to understand the parameters of that tradition, how it operated and how it was to be applied, could take part in that effort. Much time has passed, and the tradition is in tatters, assumed by and subsumed by people who simply have no clue. They do not understand the dynamics of it, the application of it, and so we have dysfunction on a wide scale.

      • Sam says:

        > that has focused on mankind’s relationship to God. That has been a Jewish undertaking. There is no parallel to it in the Christian world.

        Have you read Augustine, Aquinas, Luther, Calvin, and others before making that statement?

        >Newton also wrote on Judaeo-Christian prophecy,

        So what? He wasn’t Jewish. Neither were Leibniz, Descartes, Kant, or Hume, Mill, or Russell. Spinoza was, and rejected the whole thing. Einstein was, and didn’t believe in the religion (keep comments about “playing dice with the universe” aside, please, since they’re totally irrelevant to the topic). G3’s point still stands with regard to them: are you a greater intellect than all of them?

      • G*3 says:

        Min HaMeitzar, you completely missed both points I was making.

        Newton and Aquinas were just the first two brilliant Christians that I happened to think of. Newton’s personal beliefs are irrelevant to the point I was making, which is that there were many brilliant scholars of other religions. That a brilliant person believed X isn’t itself proof that X is true.

        In my second point I was not looking for answers. I was not asking you whether the Torah was given on Har Sinia. I was pointing out that these are empirical questions. Either it is true that the Torah was given on Har Sinai, or it is not true that the Torah was given on Har Sinai. Frustration with a chumrah-obsessed society or appreciation of Yiddishkeit’s beauty have no relevance to whether or not matan Torah happened.

        If I asked whether or not the sky is blue, you wouldn’t discuss how some blue-sky proponents frustrate you, and you therefore are thinking of dropping the notion of the sky being blue, but you came to appreciate the beauty of a blue sky and therefore you’re going to continue believing the sky is blue.

        Religion is similarly an empirical question, albeit not one so easily answered as whether the sky is blue. The acceptance or rejection of a religion should be based on whether or not one thinks its claims are true, not on whether it is frustrating or beautiful.

      • shim (Shimon) says:

        I think you are maybe misunderstanding what rabbonim are. The “rabbis” we have today are not actually Rabbonim they do not have true Smichah only a academic test that is referred to by the same name.

    • OTD says:

      I won’t say who “Min HaMeitzar” is.

    • Sam says:

      Min Hameitzar, that’s extraordinarily naive. “Rav Soloveitchik was smarter than EVERYBODY, and he believed!” Here’s a simple question: did he write a defense of religious belief and OJ in particular–by which I mean an explanation of why he believes in it to begin with–that you find convincing? That’s all that matters, and the answer is no.

      Besides which, it’s just a silly game. Bertrand Russell almost certainly knew more about “the gamut of human knowledge” than Rav Soloveitchik did, and was an atheist. The Rav was surely an unusually intelligent person, but there are other equally intelligent people out there, and it’s living in a fantasy land to think he was magical or “transcendent.” If he had a rational basis for his beliefs, he could have made it accessible to others.

    • Eli says:

      Thanks for the reply, Min. The truth is not subjective nor entirely unknowable for humans, even dumb ones. You can examine and analyze what someone says to see if it makes sense, to see if it’s consistent with other facts you know to be true. Thinking about the truth is not like following an inclination to eat ice cream. You don’t have to rely exclusively on what someone else tells you is true.
      And I share your high estimation of J. Soloveitchik. He was hador gadol in his time. But I wouldn’t want him working on my car, and I’m reasonably certain he didn’t know enough mathematics to give an example of a principle ideal domain which was not a unique factorization domain or perform a delicate operation to remove a cancerous tumor in my skull. He was a great man, a genius for sure, but still only a man. He knew what he knew and didn’t know what he didn’t know. Just because a person is “transcendently” brilliant and highly learned doesn’t mean his views should automatically be accepted unquestioned. You like his views though you admit you don’t understand how he arrived at them (plus tons of smart people are saying, “He’s really smart!”) so you say, “His views are true.” You don’t have a objective warrant for your belief; you’re just following your personal feelings and your disposition to hero worship.

      • Min HaMeitzar says:

        Eli…one more time…it is, in my opinion, absurd that you (and most people) think that “truth” is accessible to anyone. We are so limited in so many ways…seeing if something is “consistent” with “other facts you know to be true” is such a difficult statement – how do you – you- know them to be true? And who is conducting this analysis? The great “you”?

        I have no problem with people who choose not to believe, and to live their lives as they see fit. It’s a free country. But their tortured need to justify themselves, the endless preachiness (in it’s own way equally as offensive as their believing counterparts) grates on the nerves. I use Rav Soloveichik as an example, as a kind of embodiment of a cumulative experience over hundreds of years where these issues have been thought about by many, many people, with certain conclusions having been reached. You don’t have to agree with someone because they’re smarter…but it’s not just “someone” – and these people were all much much much smarter than you. There has to be some respect, some humility, in the face of that reality. Enjoy the Big Mac, if that turns you on, but pipe down about your great visions of “truth”.

      • G*3 says:

        > it is, in my opinion, absurd that you (and most people) think that “truth” is accessible to anyone. We are so limited in so many ways

        That we cannot know anything with absolute certainty is not a license to just make stuff up.

        > You don’t have to agree with someone because they’re smarter…but it’s not just “someone” – and these people were all much much much smarter than you.

        If we were to put it to a vote of great minds, we would all be Christians.

        > Enjoy the Big Mac, if that turns you on, but pipe down about your great visions of “truth”.

        Now we’re getting somewhere. You think that no one ever rejects religion because they doubt it’s true, but only so they can live a hedonistic life and be a slave to their taavos. Now your discussion of how Judaism makes you feel makes sense. According to you, that Judaism is true is a forgone conclusion. The only question is whether one gives in to his taavos and rejects it (something that becomes more likely with each added chumrah restricting what onw can enjoy) or whether one virtuously finds the beauty in Yiddishkeit and continues to keep the mitzvos.

        My only question is where do Orthoprax people like me fit into your conception of religion?

      • Eli says:

        Everyday life is filled with counterexamples to the claim truth is inaccessible to humans. Do you also claim knowledge of G-d’s will is totally inaccessible too? Isn’t the purpose of giving us the Torah to illumine our hearts with the knowledge of G-d’s will for each one of us? The generations of scholars you mention had to examine and accept or reject the conclusions of those that went before them. That certain of these conclusions have been repeatedly affirmed over so many years contradicts your claim the truth can’t be known. It’s not about being smart: Judaism isn’t a mental exercise. It’s an activity of the heart, the soul if you like.

      • Eli says:

        Min, who’s conducting your analysis? The g8 u?

      • Sam says:

        >it is, in my opinion, absurd that you (and most people) think that “truth” is accessible to anyone.

        So, you have an idea that the Jewish sages of each generation are the only people who can be trusted for truth. That itself is a proposition that is true or false. Where did you get it from? If you got it from them, that is circular logic and worthless; you need to have some reason to accept it, of course. If you accepted it on your own, though, you are using your own reasoning to come to truths, thus nullifying your own claim. So?

        >you don’t have to agree with someone because they’re smarter…but it’s not just “someone”

        Right…it’s someone who is a hero to you, and so to whom you have deferred the necessary work on establishing your beliefs in the hopes he already did it. Having a hero is fine and all, as long as you don’t get carried away. (Have you ever heard a lubab talk about the Rebbe? “You don’t understand what the Rebbe is…”) I have intellectual heros too, but I don’t assume they are more than very intelligent and knowledgeable humans who seem to have gotten a lot right and had the guts to say it.

    • DL says:

      Richard Dawkins is smarter than you , knows more than you has acute reasoning capabilities that are transcendent. How could you with integrity choose to emulate one genious’s beliefs due to his/her intelligence, but not another’s? How do you pick? It is the Rabbis’s right to rely on his own reason/faith to reach his own conclusions.

      • Min HaMeitzar says:

        Love to have put Dawkins in a room with R’ Soloveichik – or even better, R’ S.Z. Auerbach….Dawkins would be down for the count…now c’mon guys, bring the hate, the scorn, the invective…esp. G3, to whom I say once again, everyone has the right to do what they want – but it’s a matter of choice, not some great truth you’ve discovered…I’m not starting blogs about how true Torah is, and indeed some of my best friends are Orthoprax…they just don’t feel a need to tell me how right they are.

      • JG says:

        >indeed some of my best friends are Orthoprax…they just don’t feel a need to tell me how right they are.

        Min HaMeitzar, you do realize that this is an orthoprax blog you came on in order to challenge it, right? Don’t be surprised when you get a response.

  4. Accidential Korach says:

    I am more interested in how you got to where you are. for example was it (a) archaeology (b) Biblical criticism (higher or lower or both) (c) comparative ancient Near East Religion (d) the fact that the oral law was developed by humans over time and not given to Moses at Sinai? (e) other?

    • Miami Al says:

      The thing I find most sad, is that the only concept people have of Oral Law from Sinai is this childlike idea that Moses was told a list of things to write down, and a list of things to not write down, relay them to his disciples who took notes, and relayed it. That’s a cute way to describe it to children, but not an adult approach at all.

      There is a story of Moses listening in on Rabbi Akiva’s class, not knowing what he was witnessing, until he was told it was the Torah given to him. Further, there are different opinions regarded in the Mishnah and the Gemara, if this was all passed down word for word, then there would be no debate, no minority opinions.

      A more “adult” version of this suggests that when Moses was leading the Israelites, in addition to the Torah that could be read to them (in whatever form it was at the time, we know the modern scroll traces back to Ezra, why we pretend otherwise is silly), he’d be asked questions by the elders and talk to them.

      Tracing Oral Law to Moses traces LEGITIMACY of the law, the concept of a continuing court that goes back to Moses, thereby having the legitimacy to rule on his behalf.

      What we have recorded as Oral Law is a combination of accepted practices at the time, traditions that may have pre-dated Torah, and other codification of the practices of the Jewish people.

      We have practices in the written Torah without instruction, this is used by Rabbis to explain the “proof” of the Oral Law, when it proves no such thing. It means that contemporary modern Jews have no inherit concept of these items, while the people at the time just “knew” what it was. How did they know how to tie fringes on their garment? Because the Egyptian Priests had fringes, and no doubt some of the slaves had tied them, so they were told to die one thread blue to remember Hashem instead of just doing it by wrote as in Egypt. Now, none of us would know how to tie these fringes without tradition and the Oral Law, because fringes on four cornered garments haven’t been in style for 2500 years give or take, so we rely on our custom.

      When I see the “conflicts” that goes Jews to go OTD, it’s almost comical. There are certainly issues that could give one pause, but the ones brought up are juvenile in nature, which leads me to conclude that after 13 years of private Jewish education and 1-2 years in Israel, most Jews have little better an understanding of the Bible than you would expect of a 10 year old in Sunday School.

      Reconciling evolution biology with creation has been a big push by the non-fundamentalists Christians, it isn’t hugely difficult to deal with. But if you teach fundamentalism, then as adults, they’ll realize that it’s absurd, because the fundamentalist version is childish and silly.

      • Eli says:

        … childish and silly and confining.

      • Dov Kramer says:

        Miami Al,

        I wrote about this very topic this past Shuvuos.

        http://rabbidmk.posterous.com/shuvuos-5770

        Most of these “OTD issues” seem to be the result of never growing past the (neccessary) oversimplification taught in grade school.

      • Miami Al says:

        Dov Kramer wrote, “Most of these “OTD issues” seem to be the result of never growing past the (neccessary) oversimplification taught in grade school.”

        Right, but part of the problem is that in grade school, the simplification is taught so absolutely, and there is no “later” in which to teach them. They were taught a fundamentalist simplification, which is never challenged, with the idea that they can “explore” later, well, when they explore later, they realize that the simplification they learned in 4th grade is “wrong” and begin to question everything.

        Starting in 3rd or 4th grade, we study US History. As a child, you learn “George Washington and the Cherry Tree,” by middle school, you learn more about the revolution and the fight against the King, by high school, deeper issues of trade, mercantilism, smuggler profits, etc. At no point does “George Washington and the Cherry Tree” feel like a “lie,” it’s a national myth about the founding President, a man of such strong integrity he preserved the Republic instead of establishing a new Monarchy.

        In the Yeshiva World, the kids learn the “Sunday School” level stories, then when old enough to learn more seriously, they have abandoned Tanakh and moved on to Gemara, which they are too young to comprehend as what it is (recorded legal codes) and instead learn the “game” of it.

        As a result, the first “truth” that they are exposed to is anti-Bible academics, the only question is, are the curious enough to explore this before they are married with children and “trapped,” or do they explore it later and therefore be trapped in the system.

        That’s an extremely stupid approach, when a more nuanced and mature model of education would eliminate the problem.

      • Dov Kramer says:

        Al,

        I’m not sure that most teaching those “Sunday school” oversimplifications know any better themselves. An unfortunate result of not being able to hire better-trained, smarter educators away from much higher paying professions (tuitions are already very high, as you mentioned).

        It is therefore up to the parents to fill in the stuff between the lines. The hardest part of this, I have found, is finding a way to correct the oversimplifications without causing my kids to lose respect for their teachers.

        Very few (if any) worthwhile things come easy.

        Relearning Tanach is also a serious issue, and the Yeshiva my sons go to, which uses innovative educational techniques (okay, I was a co-founder of the school) starts teaching Gemoro later than most, and focus on teaching the skills to be able to learn pesukim they haven’t yet seen on their own. My oldest son (going into 4th grade) went through all of Beraishis, and was one of 6 boys in his class to “put Beraishis on the Menahel’s desk” and get farhaired on it. When he gets together with his friends on Shabbos, they first learn a Mishna, taught by one of the boys (who learned it with his father), before they “play.” His brother (going into 3rd grade) learns Pirkay Avos with 3 other boys, taught by one of the fathers, before they play.

        When they (or their older sisters) say over a D’var torah at the Shabbos table, their pesky father asks them questions to make sure they understand what they said, and the ramifications of what they said (hopefully on their level). B”H I can see their critical thinking process developing, and they ask really good questions (and are complimented for doing so). Hopefully, with HaShem’s help, they will have a fuller perspective about things.

      • Miami Al says:

        Dov Kramer,

        That’s wonderful, and a great way to educate your children, and in line with the advice from our sages. IIRC, it’s Tanakh starting at 5, Mishnah at 10, Gemara at 15 (from Pirkei Avot). That, plus practical Halacha from either Shulchan Aruch or Mishneh Beruah (easier to comprehend) would give children a pretty solid textual education.

        Add a Jewish Philosophy Track at the high school level, which would include practical issues (20th and 21st Century rulings) and Jewish History at all levels (real history, not whitewashed “fake” stuff that will undermine their beliefs if they ever leran more), and you have a pretty solid religious education.

        Some of the costs are getting qualified teachers, and some of the costs of Yeshiva are providing a secular education within a “religious” institution, some of the costs are a socialistic system that results in all schools charging about the same and taking all regardless of ability to pay, and some is high administrative overhead of schools that are more focused on administrative availability to parents than maximizing educational dollars.

        If a good solid well rounded Jewish education was important to parents, the schools would provide it. Alas, they are more concerned with a “good education” (meaning secular education), and care little about the religious instruction beyond it existing, so you have the status quo.

  5. baltlaw says:

    It’s easy to eat a piece of Matza on Pesach, but to my way of thinking, the easiest positive mitzvah of them all is to hear the Shofar blown. All you have to do is show up. (to paraphrase Woody Allen).
    We are about to read Parashat Ve’Etchanan, which contains, of course, the 10 Commandments and the Shema. These are the ways we proclaim our faith.
    Belief in God is, to me, the hardest Mitzvah. You can hear the Shofar and taste the Matza. You can feel the crunch of the matza and you can touch the Shofar to your lips.
    But, “belief” is very much an emotion.
    You are struggling and the struggle is understandable. Perhaps someday soon you’ll return to the derech and maybe that Shuvah will occur this Rosh Hashana. Perhaps not.
    Hashiveinu Hashem Alecha, V’nashuva Chadesh Yamenu K’kedem.

    • Dov Kramer says:

      I never liked the word “belief.”

      “Emunah” comes from the word “emes,” truth (Radak says the real word is “amenes,” but the “nun” was removed to make it easier to say, much like “benas” became “bas”).

      Moshe was a “ne’eman” because he was loyal, stayed true to his ideals.

      “Emunah” is not an emotion, it is a commitment to stay true to yourself, true to your well-thought out conclusions (if they’re not well thought out, you’re not being true to yourself).

      Wheras “bitachon” refers to trusting that G-d (or whomever you are trusting) will do what He/She is supposed to/promised to do, “emunah” refers to our loyalty in doing what we are supposed to do.

      If they were just emotions, they would have little objective value. Emotions are there to help us act upon what we think we should do, not decide what we should do.

  6. Joseph Mullen says:

    “it took one hundred years until The Civil Rights Act was passed and even today, there are difficulties with that gracious and overdue act.”

    Difficulties? You mean that this law promotes reverse racism against whites? Or that it causes employers to hire people who they know will ultimately bring down the level of their company? Kindly illustrate for us what exactly are the difficulties in Title VII.

    When you utter vapid liberal platitudes like this it makes me think your atheism is not the result of serious thinking as you claim, but rather because other left-wingers are atheists, so you may as well just join the club.

    • ExYid says:

      Relax there, buddy. I think you missed the point. No one actually cares about the substance of his analogy. He was trying to build up to his last sentence about freedom, which you can probably understand without the emotive content. Smart guy like you. (Throwing around chains of $10 words like “vapid liberal platitudes” – adjusted for inflation – is always indicative of greater intellect.)

    • Eli says:

      It’s not racism. It’s an attempt to rectify or ameliorate the historical effects of centuries of slavery which are actually racist. When someone utters trite conservative slogans it makes me think they need to start thinking for themselves and not just mega-dittoing the views of others.

  7. Suzanne says:

    Although you may feel alone in that you are compelled to talk the party line about God in order to keep your job, I think there are many “congregants” and members of such communities as yours that also feel that compulsion to affirm or at least entertain belief in God among others of their community. Not because they truly have that belief, but because like yourself they are afraid of what would happen if they really spoke of their doubts. Living in a non-orthodox environment is truly freeing in that sense, since no one really expects firm commitment to either practice or ideology. I grew up Modern Orthodox, but no longer live in that community and slowly I am learning to be less afraid of sharing my questions and doubts.

  8. ExYid says:

    I’m a little confused about your stance on educating you son. Presumably you think you found the truth (that there is no Capitalized Truth). How, then, can you genuinely want for your son to grow up in an intellectually limited corner of the universe? I posted on your first article what I thought was a convincing argument to keep people in the dark – to spare them the uncertainty of not knowing, as you would spare your son. But that now seems dishonest to me. I would seek the truth, as I am compelled to do. Does this mean that I am dedicated to a goal that I don’t think is worthy for others? How can I, in good conscience argue that I may seek what no one else should? I suppose what I really want to know is: how can you?

    My natural skepticism inclines me to make this argument into a sort of axiomatic progression. My first axiom would have to be that truth (or Truth, as most would like to be believe exists – and I think you understand the distinction) is worth seeking. But even that does not seem so evident. I know that I want it, and I know that I would enjoy knowing the knowledge if it came, but is there any real reason to pursue it over blessed ignorance, over the avoidance of pain or the deadening bliss of drugs? What sort of principles are there on which to base any sort of faith? In a way, I envy your son, and my own ignorant past. I was certainly more happy back then. Yet, now, I realize that I simultaneously crave its realization once again, and yet I know that I ought not. On the surface, non-belief has the same problems as belief. And yet, these problems are a product of a non-belief standpoint. I have so far only gauged acceptability on intuitive criteria. Yet now, my intuition pulls me in two. Your problem puts my own in a new light, and I think that’s why I’m drawn to your blog. If I had to quit rambling and ask you directly what I’m trying to find, I think it wold best be phrased as follows: How can you be so certain in your assertion of non-belief? How do you live with yourself in your situation? And how do you justify not teaching your son to think like you if you think that you are teaching him lies?

    Thanks for the compelling topic.

  9. Ephraim says:

    “Min Hameitzar, that’s extraordinarily naive. “Rav Soloveitchik was smarter than EVERYBODY, and he believed!” Here’s a simple question: did he write a defense of religious belief and OJ in particular–by which I mean an explanation of why he believes in it to begin with–that you find convincing? That’s all that matters, and the answer is no. ”

    Indeed that type of apologetics seems to have been outside the Rav’s style. You’re talking about someone who breathed Torah from the cradle. When the Rav wrote about the Torah experience he wrote descriptively, not defensively. In a (sort of) straightforward way he defined and described what “is”. He wrote about the “hows” and “whats” , but didn’t (at least not usually) write about the “whys”.
    Here’s the famous quote from Lonely Man of Faith:
    “I have never been seriously troubled by the problem of the Biblical doctrine of creation vis-a-vis the scientific story of evolution at both the cosmic and the organic levels, nor have I been perturbed by the confrontation of the mechanistic interpretation of the human mind with the Biblical spiritual concept of man. I have not been perplexed by the impossibility of fitting the mystery of revelation into the framework of historical empiricism. Moreover, I have not even been troubled by the theories of Biblical criticism which contradict the very foundations upon which the sanctity and integrity of the Scriptures rest.”

    I suppose a crude analogy would be for you to write a defense against my notion that Rembrandt (or Beethoven, or Shakespeare etc..) was a talentless hack. I suppose such a defense could be written convincingly. But for someone who doesn’t experience the painting as something wonderful and inspiring, such an apologetic is irrelevant. And one who does have that experience will not have a need for such apologetics. (Though such apologetics may deepen his appreciation and provide an intellectual component to an otherwise purely aesthestic experience.)
    I just read an autobiographical excerpt from one of the noted 19th century maskilim. He writes that all he got from his Talmudic studies was an intellectual thrill. It was the logic that enthralled him; the underlying content meant nothing to him. Of course, someone like that would flee the bais medrash as soon as he had the opportunity. Someone who truly enjoys learning, whether it’s the dialectic or simple psak will not leave so easily. He may be tortured by doubt, but he’s going do his damndest to overcome his doubts. Because he doesn’t want to lose what he treasures. Someone like that will study and struggle to justify the Torah lifestyle that is so dear to him. Someone who starts with the notion that it’s all nonsense, or that everything must be proven scientifically, is unlikely to try that hard. He will fulfill his obligation of inquiry with the minimum requirements of Moreh Nevuchim, Kuzari and a tape by Avigdor Miller and then head for the exit.

    • Sam says:

      >Indeed that type of apologetics seems to have been outside the Rav’s style.

      >When the Rav wrote about the Torah experience he wrote descriptively, not defensively.

      Yes, but the comment I was responding to was that the Rav was more knowledgeable and more reasoned than others and yet a believer, so we should all rely on his intelligence to have been right and shouldn’t question his judgment in the matter.

      But this is precisely the point. For those who are concerned with the issues the Rav wasn’t concerned with, how relevant can his intelligence be if he didn’t care to turn it to these issues? Some of us are troubled by those issues, which do venture into the realm of empirical fact.

    • Min HaMeitzar says:

      Precisely…Rav Soloveichik felt no need for apologetics. The point about belief, and challenges to it, and so on, is that it is all so subjective. As I’ve written to others in this discussion, the search for “truth” is really colored by so many elements within our psychological makeup that the ludicrous attempts to “prove religion wrong” are really flimsy bandages that are created to mask deeper issues…of hurt, of loneliness, on and on. Orthoprax Rabbi is married. His “search for truth” is going, quite possibly, to lose him that marriage. Umm…love….”truth”….love…”truth”…I’ll take love….anytime.

      • Eli says:

        Take truth and live and grow.

      • tayqoo says:

        are you saying that since “it is all so subjective” we might as well believe the unbelievable? i think that you are grasping at straws. been there, done that.

      • David says:

        I guess most of the skeptics decided that they’d have to go with truth. Remember that scene in The Matrix? You chose the pill you wanted to swallow. I understand why you’d want to do that, but some of us can’t stop our gag reflexes.

      • Eli says:

        The force of skeptical arguments is almost always overestimated. For example solipcism is irrefutable; it’s also irrelevant.

      • Sam says:

        > The point about belief, and challenges to it, and so on, is that it is all so subjective.

        Not really, and I bet you don’t act in accordance with that on a daily basis. You constantly accept facts as true or reject them based on what seems true to you. Moreover, I imagine the relativistic implications of what you are saying would be abhorrent to you in another instance. Yet in one particular area of life, you find it necessary to shut off rational faculties and claim that “everything is subjective” so you can believe what you want. It’s an empty and meaningless claim.

        Either the religion’s claims can be shown to be true, or not. Hiding behind “everything is subjective” does not demonstrate much confidence in the former.

        >the search for “truth” is really colored by so many elements within our psychological makeup

        If the search for truth were impossible, then what you are writing would be as meaningless babble as anything else, and you should know that. You don’t believe that, though–you undoubtedly think that YOU have a good point. Why? Because in reality, you believe in rationality and the accessibility of truth.

    • Sam says:

      Let me adapt Ephraim’s analogy to demonstrate the point, since it seems to be slipping by Min HaMeitzar.

      I love Shakespeare. I would be deeply pained if it were true–as some claim–that Shakespeare did not really exist or did not really write his works, and they were written by a collection of others. I’m very glad and relieved that I have read and heard from authoritative scholars that this idea is wrong.

      But the reason I get to believe that Shakespeare existed is not that I find his work beautiful and therefore don’t need to consider anything else. Sure, I might not have WANTED to look at the evidence if the evidence went the other way, but I could not legitimately say “I have felt what it is like to read Shakespeare, and so I have no need for evidence! Truth is subjective anyway, and these scholars just don’t get what it’s like to appreciate Shakespeare!”

      Same for if someone somehow scientifically showed that listening to Beethoven is bad for you. I would be completely crushed and incredulous. But that is not the reason I get to disbelieve that proposition; it’s ultimately because of the reason and evidence.

  10. Min HaMeitzar says:

    G3…I don’t really care about people’s choices. They are choices that they are free to make, and I don’t condemn them for making them. I absolutely do NOT believe that people choose to follow a non-religious lifestyle for the sake of a Big Mac, or “taavos”, although that may be a factor for some. I think that these choices are deeply seated in the psyche…emotional deprivation, anger, and a host of other things may contribute to that choice. I like people of all sorts, non-Jews, Jews, young, old….as long as they are real about who they are and what they do. I think Rav Soloveichik was very real, and I think that Orthoprax rabbi needs to get real…so if I’d meet you, in all your glorious orthopraxness, and you’d want a Big Mac for lunch, I’d happily join you ( I’ll have a Coke 🙂 )…

    • G*3 says:

      > I absolutely do NOT believe that people choose to follow a non-religious lifestyle for the sake of a Big Mac, or “taavos”, although that may be a factor for some. I think that these choices are deeply seated in the psyche…emotional deprivation, anger, and a host of other things may contribute to that choice.

      I see. It’s not that atheists are weak-willed and can’t control themselves. It’s that atheism is sort of a disorder, brought on by “emotional deprivation, anger,” etc.

      For the record, I had a pretty good childhood, and I’m not angry. In fact I’m probably one the most easy-going people you’d ever meet.

      As for “a host of other things,” sure there’s lots of stuff that effect our decisions. But those same factors equally influence the decision to stay frum.

      Most importantly, people do not choose what to believe. They may choose a religious or non-religious lifestyle, but actual beliefs are determined by what one considers to be true. To reuse my earlier example, one cannot choose to believe that the sky is green. Similarly, belief in God and Yiddishkeit is not a choice like choosing which flavor of ice cream to have. Either one thinks God is real, or one doesn’t.

      This is my main point, and one that you continue to ignore. Beliefs about empirical facts are not subject to choices nor are the facts influenced by the way we feel about them.

      • Dov Kramer says:

        >>Most importantly, people do not choose what to believe<>…belief in God and Yiddishkeit is not a choice like choosing which flavor of ice cream to have. Either one thinks God is real, or one doesn’t.<<

        No. It's not an "all or nothing" conclusion. Very few decisions in life are based on absolute knowledge. It comes down to how likely you think something is.

        For example, no one who invests can know for sure that they will make $$ on a particular investment (or how much). Whether they invest, and how much they'll invest, depends on their estimation of the likelihood o making $$ (taking the risk/reward ration into account as well).

        There is no reason we have to have absolute knowledge of the Creator's existance before committing ourselves to finding out what He/She wants/expects from us. How likely His/Her existance has to be estimated to be before making such a committment varies from person to person (and here the percentages are greatly affected by the perception of what might be required), but to say it's either yes or no, I don't buy it.

        The question isn't if I think G-d exists, but how likely do I think it is that He/She exists, and ay what point is the likelihood enough to warrent committment.

  11. Noam says:

    so what seems to be really odd is that your entire conviction isn’t based on logic or reason. You feel that God does not exist… in that sense atheism is your true religion. You’re feeling may be based on various questions you may have or thoughts that have occurred to you… but in the end you choose to believe what you feel… and that feeling tells you that God doesn’t exist.
    This is fine, and there is nothing wrong with this, but please don’t pretend that you have come to your conclusion through thorough logical analysis… in the end you’re doing what you feel… same as anyone else.

  12. Min HaMeitzar says:

    Tayqoo…don’t believe anything you don’t believe in…behave honorably, do what you feel is right…but keep your mind open, and your respect for everyone, religious or otherwise, intact….

  13. Puzzled says:

    How bizarre, this “JB was the only intelligent person to ever think about religion” story. I cannot think of a better way to establish authoritarian religion than to tell people they are too dumb to disagree with your leader. By the way, what’s with, well, all the orthodox people in the world outside of the MO/YU derech? Why are they also stupid enough to disagree with the Rav?

    I saw a blog post yesterday about JB, actually. It went through the math – what portion of the world is Jewish? Orthodox? Brisker? American? MO? Interestingly, it turns out that the Rav was a member of a group that consists of, I think it was, .0001% of the world – and that just happens to be the group he was born into, in a sense – he wasn’t born in America, but his father immigrated, etc. Seems a little odd that this is entirely the result of reasoned thought, doesn’t it?

  14. Zak says:

    “I think that these choices are deeply seated in the psyche…emotional deprivation, anger, and a host of other things may contribute to that choice.”

    That sounds like a better explanation for the desperate and willfully blind defenses people continue to erect in favor of a God notion that has long since been conclusively
    discredited by some of the worlds greatest minds utilizing epistemological modalities so astonishingly advanced, that they would, dare I say, leave someone like the Rav in utter slack-jawed confoundment.

  15. Ephraim says:

    “But this is precisely the point. For those who are concerned with the issues the Rav wasn’t concerned with, how relevant can his intelligence be if he didn’t care to turn it to these issues? Some of us are troubled by those issues, which do venture into the realm of empirical fact.”

    I agree with you 99%. This is the major reason why I was upset over the Slifkin affair. There were elements of the opposition whom I sympathized with. But the world doesn’t revolve around me. Not one of the many distinguished rabbis who attacked Slifkin offered any solution to the problems Slifkin addressed. They totally ignored those who were struggling with these issues and they sought to silence others who tried find solutions to these same questions. I’m talking about huge geonim who rarely left the bais medrash. Of course they would scoff at Slifkin. They’re flying in the stratosphere while Slifkin is teaching people to stand up!
    This is why any program which takes the rational approach that Slifkin, Schroeder and others have taken must include a parallel track which seeks to inculcate a feeling of the beauty of Torah. The heart must be engaged as well. The What and How must be taught, not just the Why.

  16. Baal Habos says:

    >Who made Hashem? Frankly, who cares?

    Not that Atheism is my cup of tea, but frankly the question misses the point. If there’s a God, indeed I don’t care who created him. If I believe in TMS, then I know there is a God and thus I don’t care. But without TMS, without revelation, I don’t even know that there is a God, and thus the ‘who made Hashem’ is used to question the premise of said God.

  17. Baal Habos says:

    >You are not more intelligent, to put it mildly, than Rav Soloveichik, than R’ Akiva Eiger, and so on, all of whom were quite happy to remain believers.

    Just curious. How do you know? (I’m not talking about me, but maybe our erstwhile Rabbi has an IQ of 170?)

  18. JG says:

    To those insisting that truth is subjective when it comes to religious discussions and that we cannot rationally inquire into these matters:

    I would just like you to know that, logically speaking, your point directly implies that God is a tuna sandwich. What kind of God is that to worship? Is that really the logical road you want to go down?

    [Pause.]

    [“Huh? How does what I said imply that at all?”]

    To quote Dan Dennett:

    “Oh, do you want the net [of rationality] up for my returns, but not for your serves?

    Either way the net stays up, or it stays down. If the net is down there are no rules and anybody can say anything, a mug’s game if there ever was one. I have been giving you the benefit of the assumption that you would not waste your own time or mine by playing with the net down.”

  19. YH says:

    This is more a response to all of your posts than to this specific one.
    Although I have never had a congregation, I too have semicha and experience teaching in Yeshiva. I respect how honestly you have searched and confronted the big questions, something I strive to do as well, though I have reached quite different conclusions. What I can not respect is your assertion that you can continue to serve as a community Rabbi. Put simply, you are lying to these people (and your wife and kids). This is in no way like a plumber. These people have put their trust in you and you are deceiving them. I can only assume that if they found out who you really are they would be quite upset – perhaps even devastated. My objection to what you are doing is not “frum” or based on Torah. Rather, on a basic human level I fail to see any way to justify your actions.

    Be a man. Step down.

    • amldc says:

      My guess is that this blog is his first attempt at stepping down. I feel for him, I really do.

    • If I were to follow through with your suggestion, what about my congregants? I have helped many through good and bad times and they value our relationships. Should I just leave them in the lurch? You aren’t the first to note that if my congregants find out they would be devastated, but how would they find out? I don’t ever expose my inner held beliefs to them – my position are just that, my own thoughts. Until there is a mind reading machine, I don’t see how they come out. Is it because of this website? If so, I don’t see how that changes anything. I have put in place various mechanisms to ensure my anonymity. Of course, anything is possible but that doesn’t mean it likely. Is this project any different than a personal diary recording my thoughts?
      In the end it comes down to the fact I am providing a valuable service to my shul, in return they pay me and seem to think that worthwhile. If anything stepping down is not “being a man” it is running away, it is being a coward, it is leaving my congregants, my friends, alone.

      • Simcha says:

        What if you became convinced that the Christians had a point and salvation were available only through Jesus? If you didn’t tell your congregation and continued fulfilling all your responsibilities as you had for years, would that be OK?
        When you are a pulpit rabbi, it’s not just about your ability to help the members of your congregation, skeptics and believers alike, it’s also got to be about being able to model a more ideal Jewish life.

      • YH says:

        I think you misunderstood my comment. The problem is not that your congregation might find out. What I mean to point out is that their reaction, if they theoretically did find out, is an indication of the magnitude of your lie and betrayal.
        As talented a Rabbi as you may be, I’m sure you are replaceable. You will not be the first Rabbi to retire.

      • guy says:

        “If anything stepping down is not “being a man” it is running away, it is being a coward, it is leaving my congregants, my friends, alone.”

        this is one of the most delusional statements i have ever heard-

    • Only a Jew says:

      Where have I heard that before?

  20. tayqoo says:

    min hameitzar. this is what you wrote…
    Min HaMeitzar says:
    July 14, 2010 at 12:39 pm
    Precisely…Rav Soloveichik felt no need for apologetics. The point about belief, and challenges to it, and so on, is that it is all so subjective. As I’ve written to others in this discussion, the search for “truth” is really colored by so many elements within our psychological makeup that the ludicrous attempts to “prove religion wrong” are really flimsy bandages that are created to mask deeper issues…of hurt, of loneliness, on and on. Orthoprax Rabbi is married. His “search for truth” is going, quite possibly, to lose him that marriage. Umm…love….”truth”….love…”truth”…I’ll take love….anytime.

    this what i asked….
    tayqoo says:
    July 14, 2010 at 1:45 pm
    are you saying that since “it is all so subjective” we might as well believe the unbelievable? i think that you are grasping at straws. been there, done that.

    this is what you answered…
    Min HaMeitzar says:
    July 14, 2010 at 2:07 pm
    Tayqoo…don’t believe anything you don’t believe in…behave honorably, do what you feel is right…but keep your mind open, and your respect for everyone, religious or otherwise, intact….
    ——————————————-
    how is that an answer? i wasn’t asking for your advice.

  21. tayqoo says:

    Dov Kramer says:
    July 15, 2010 at 6:43 am
    >>Most importantly, people do not choose what to believe…belief in God and Yiddishkeit is not a choice like choosing which flavor of ice cream to have. Either one thinks God is real, or one doesn’t.<<

    No. It's not an "all or nothing" conclusion. Very few decisions in life are based on absolute knowledge. It comes down to how likely you think something is.

    For example, no one who invests can know for sure that they will make $$ on a particular investment (or how much). Whether they invest, and how much they'll invest, depends on their estimation of the likelihood o making $$ (taking the risk/reward ration into account as well).

    There is no reason we have to have absolute knowledge of the Creator's existance before committing ourselves to finding out what He/She wants/expects from us. How likely His/Her existance has to be estimated to be before making such a committment varies from person to person (and here the percentages are greatly affected by the perception of what might be required), but to say it's either yes or no, I don't buy it.

    The question isn't if I think G-d exists, but how likely do I think it is that He/She exists, and ay what point is the likelihood enough to warrent committment.
    ***************************************************************************

    it's a long way from belief in god (whatever that is supposed to mean) to belief in "yiddishkeit". even if my mind or my gut concludes that god (?) probably exists how do get you get to a "yiddishkeit" that seems to require belief in the improbable (to the point of the incredible).
    i'm not interested in a debate. there is nothing more that i would want (and i'm sure others too) than the ability to believe again. but belief is not something that can be turned on and off at will. just because i'd like to believe in something doesn't make it believable (tms, moshiach, olam haba, tchiyat hametim etc). that's what makes this whole discussion painful and frustrating. i'd like "god" and "yiddishkeit" to be as real as gravity. sadly, i don't think that ANYONE had/has answers just opinions. everyone chooses or creates an opinion that floats their boat.

    • Miami Al says:

      If you look at the Kiruv world, you’ll see some meaningless propaganda, but also a non-fundamentalist and rational approach to Yiddishkeit.

      I remember an article on Aish.com dealing with Baalei Teshuva, and about when she “believed in God.” She talked about Shabbat Candles, Shabbat, Kashrut, and Taharat Hamishpacha, and as she was practices, she found an inner beauty in it, and through doing so, found God in it.

      The whole TMS -> belief -> practice just seems so juvenile to me, a great way to teach children, but a lousy way to keep adults engaged, because it isn’t fascinating, it’s a serious of “leaps of faith” that quite frankly aren’t so important to an adult.

      For a BT, when you observe Torah, you live your life within a certain lifestyle. Either that lifestyle uplifts you, guides you, and shows you a wonderful life, such that you believe your “new” life is better than your previous one, and through that you see a Devine influence, or it’s worse and you leave. For those that grew up in the system, if you see that Yiddishkeit and it’s elements uplift you and give you a “heaven on earth life,” then you see a sense of truth in the Mitzvot. If you feel that Yiddishkeit holds you back and makes your life worse, then you ask, “What sort of God would demand this of me?” If you don’t want to be in the system, it’s pretty easy to poke holes in the juvenile theology that most possess, if you want to be in the system, they are completely irrelevant.

      If you hate keeping Mitzvot but do so out of belief, you’re resent it. Better to find a way to keep Mitzvot that you love the overall equation, even if you have issues with individual ones.

      If you have a wonderful spouse and a loving environment for children, you’ll LOVE being Jewish. If you have a spiteful wife, financial problems, and a level of obligations that you can’t handle, then you are going to HATE being Jewish (you’d also hate being Reform, Atheist, Catholic, or Protestant as well).

      The question is, are your problems Yiddishkeit, or are your problems more mundane, and Yiddishkeit is easy to last out at.

      • Dov Kramer says:

        Beautiful comment, Al.

        From my perspective, it starts with the rational search; the experiences that come after trying to live a life of Torah and Mitzvos validate the conclusions reached.

    • Miami Al says:

      If you look at the Kiruv world, you’ll see some meaningless propaganda, but also a non-fundamentalist and rational approach to Yiddishkeit.

      I remember an article on Aish.com dealing with Baalei Teshuva, and about when she “believed in God.” She talked about Shabbat Candles, Shabbat, Kashrut, and Taharat Hamishpacha, and as she was practices, she found an inner beauty in it, and through doing so, found God in it.

      The whole TMS -> belief -> practice just seems so juvenile to me, a great way to teach children, but a lousy way to keep adults engaged, because it isn’t fascinating, it’s a serious of “leaps of faith” that quite frankly aren’t so important to an adult.

      For a BT, when you observe Torah, you live your life within a certain lifestyle. Either that lifestyle uplifts you, guides you, and shows you a wonderful life, such that you believe your “new” life is better than your previous one, and through that you see a Devine influence, or it’s worse and you leave. For those that grew up in the system, if you see that Yiddishkeit and it’s elements uplift you and give you a “heaven on earth life,” then you see a sense of truth in the Mitzvot. If you feel that Yiddishkeit holds you back and makes your life worse, then you ask, “What sort of God would demand this of me?” If you don’t want to be in the system, it’s pretty easy to poke holes in the juvenile theology that most possess, if you want to be in the system, they are completely irrelevant.

      If you hate keeping Mitzvot but do so out of belief, you’re resent it. Better to find a way to keep Mitzvot that you love the overall equation, even if you have issues with individual ones.

      If you have a wonderful spouse and a loving environment for children, you’ll LOVE being Jewish. If you have a spiteful wife, financial problems, and a level of obligations that you can’t handle, then you are going to HATE being Jewish (you’d also hate being Reform, Atheist, Catholic, or Protestant as well).

      The question is, are your problems Yiddishkeit, or are your problems more mundane, and Yiddishkeit is easy to last out at. Yiddishkeit offers a wonderful life if practiced in an enjoyable manner, but misery if practices in a competitive Chumrot manner.

      I found that stepping back from stuff that frustrated me, I was able to carve out a wonderful life for me and my family within the bounds of Yiddishkeit, and I am very happy with my life. For those that have not been able to do so, the problem is you and your life, not Yiddishkeit, and blaming God, the Rabbis, the Rosh Yeshivot, etc., is juvenile, you’re an adult, take responsibility for your life.

      If you hate the yoke of heaven but “believe,” you will constantly try to undermine that belief to get out, which sucks, just accept that you reject the yoke of mitzvot and leave. Otherwise, you will either spend your life trying to reject your beliefs while following them (a path to misery), or otherwise approach things very bitterly.

      • tayqoo says:

        i think that what we have here is a failure to communicate.

        how was your comment a response to mine?

        what did your comment even mean?

      • G*3 says:

        > If you have a wonderful spouse and a loving environment for children, you’ll LOVE being Jewish. If you have a spiteful wife, financial problems, and a level of obligations that you can’t handle, then you are going to HATE being Jewish

        I have a wonderful spouse, a great environment for my kids, no financial problems, no problem with my level of obligation – altogether, a good life. I don’t love being Jewish (by which I assume you mean frum) and I don’t hate it. I just don’t think any of Judaism’s mythos describes reality.

        That an idea makes you feel good doesn’t mean it’s true. That an idea makes you feel bad doesn’t mean it’s false.

        > The whole TMS -> belief -> practice just seems so juvenile to me

        As opposed to, “This lifestyle makes me feel good, therefore I see the Divine / this lifestyle makes me feel bad, therefore God couldn’t have commanded I do this?” Emotions do not determine empirical reality, nor are they a good way of judging what is real.

      • Miami Al says:

        G*3,

        Being Jewish = Practicing Judaism in a serious manner… Shomer Mitzvot… Being a Jew means your mom is a Jew or you converted. 🙂

        Here is what I meant… a series of rules and regulations that seemingly make no sense come together in a package that creates a wonderful life enhancing combination. That shows some sort of remarkable string of luck, or divine influence, and not sure that there is a huge difference there.

        By juvenile I mean, hanging the belief structure on a literal revelation by which the Oral Law was LITERALLY transmitted completely to Moses (who kept it hidden so it could be revealed over the centuries), and suggesting that without this claim there is no Judaism.

        When describing a god of any kind, you can make any sort of ridiculous claim you want, and it’s not really fallsifiable or provable, it’s just your claim about your invisible friend. When making claims about humans (including the notion that all was told to Moses yet he wouldn’t recognize Rabbi Akiva’a lecture), it gets a little more silly.

        Empirical reality? For the bulk of the last 1500 years, the Jewish people have seen an Sabbath/Kashrut observant lifestyle as the “ideal” Jewish behavior, and practiced these things to various levels. As a result, that forms the core of the Jewish Nationhood/Cultural experience, and if you want to be a part of that experience and continue it, you have to fall somewhere within that spectrum.

        That’s the empirical truth.

    • Dov Kramer says:

      tayqoo-

      >>it’s a long way from belief in god (whatever that is supposed to mean) to belief in “yiddishkeit”. <> belief is not something that can be turned on and off at will<>everyone chooses or creates an opinion that floats their boat<<

      No. They choose how much time and effort to put into forming an informed opinion, and must try being as objective as possible if they want that "opinion" to be as accurate as possible.

      • Dov Kramer says:

        For some reason, my last reply was truncated (in the middle).

        Take two for tayqoo-

        >>it’s a long way from belief in god (whatever that is supposed to mean) to belief in “yiddishkeit”. <>belief is not something that can be turned on and off at will<>everyone chooses or creates an opinion that floats their boat<<

        No. They choose how much time and effort to put into forming an informed opinion, and must try being as objective as possible if they want that "opinion" to be as accurate as possible.

      • Dov Kramer says:

        Must be my way of “quoting.” Take three:

        “it’s a long way from belief in god (whatever that is supposed to mean) to belief in “yiddishkeit”. ”

        100%. But that is the first step of the process. The next step, after concluding that it is likely that there is a Creator, is trying to figure out what He/She expects from us.

        “belief is not something that can be turned on and off at will”

        See my comments above about “belief.” We don’t choose what we “believe;” we choose the process through which we come to our conclusions.

        “everyone chooses or creates an opinion that floats their boat”

        No. They choose how much time and effort to put into forming an informed opinion, and whether to try being as objective as possible in reaching that conclusion.

    • G*3 says:

      > No. It’s not an “all or nothing” conclusion. Very few decisions in life are based on absolute knowledge. It comes down to how likely you think something is.

      Yes, true. I did not mean to imply that one must have absolute knowledge. The point I am trying (and apparently, failing) to make is that belief cannot be turned on and off like flicking a light switch. Either you think God (probably) exists (because it seems likely to you) or you think He (probably) doesn’t exist, (because it doesn’t seem likely to you). It’s not a choice.

      Being religious is a choice, and one can choose to be religious even if one thinks it’s unlikely God exists, just as one can choose to invest in a stock one thinks is unlikely to do well.

  22. tayqoo says:

    “100%. But that is the first step of the process. The next step, after concluding that it is likely that there is a Creator, is trying to figure out what He/She expects from us.”

    fine i’m all ears. let’s figure it out together.

    i’m still more orthoprax than not. please explain to me why i should continue.

    • Dov Kramer says:

      >>fine i’m all ears. let’s figure it out together.<<

      From which point in the process? Are we starting with there being a Creator and trying to figure out what He/She wants/expects from us?

      • tayqoo says:

        [From which point in the process? Are we starting with there being a Creator and trying to figure out what He/She wants/expects from us?]

        god doesn’t need to be the creator and the creator doesn’t need to be god. that is all besides the point.

        yes, let’s try to figure out what “god” wants or expects from us (if anything) and how we know that.

        it’s your move.

      • Dov Kramer says:

        >>yes, let’s try to figure out what “god” wants or expects from us (if anything) and how we know that.<<

        Okay. The next step would seem to be trying to figure out what this "god" is all about, so that we can try figuring out what "it" expects/wants of us. Let's go through the possibilities.

        1) The Creator left the scene, and therefore wants/expects nothing of us.

        2) The Creator is still involved with His/Her creation, but is not a rational, logical being, so it would be impossible to figure out what He/She wants (nor would there be a reason to want to fulfill it) unless there was a direct communication telling us this information.

        3) The Creator is still involved, is a rational/logical being, and has a reason/ plan for creation.

        There may be subsets of these three possibilities, but have I left out any major, realistic possibilities?

  23. Accidential Korach says:

    Miami Al and others –

    “What we have recorded as Oral Law is a combination of accepted practices at the time, traditions that may have pre-dated Torah, and other codification of the practices of the Jewish people.”

    Clearly you agree that there is no [or perhaps only a non-identifiable whisper of] Sinaitic Oral Law.

    This was my point.

    I agree that there is a more nuanced way to understand the collection of writings that is understood to make up the Oral Law (as implied by my statement that “the oral law was developed by humans over time” ).

    Does this nuanced understanding help people to remain within the Orthodox fold? The answer to this is “no”.

    My question to the Orthoprax Rabbi (amongst the other questions posed) is was this part of (or all of) the tipping point?

    As an aside – all legal systems have some form of oral law developed around their practices. You can call this etiquette or culture.

    • Dov Kramer says:

      Korach-

      The description you use does not negate the possibility of a core originating at Sinai, or of the system for future law development originating at Sinai.

      Traditional Judaism (or OJ, if you prefer) does not say that every law originated at Sinai, or even that most laws did (certainly not that customs did).

    • Miami Al says:

      Accidential Korach,

      I think that there is an Oral Tradition that dates back to the original founding of the Jewish people, the father of the Israelites being referred to as Moses, an Egyptian Prince that started his offshoot with some slaves that he escaped with, took with, etc.

      I think that the numbers we are given are largely exaggerated, but I don’t think any of that takes away from the history of the Jewish people. An early monotheistic faith, a people that worshiped the God of Israel instead of Ra, with many powerful symbols for everyone to do.

      The most remarkable thing introduced by Judaism isn’t monotheism, it’s the democratization of religion. While early Israelite religion had a hereditary priesthood that offered requests with God, many of the commandments were for EVERYONE, and that’s pretty unique. Shabbat, Shemita, Yovel, Sukkot, Pesach (offering, not the later seder), Tzitzit, Milah, etc., these were all things for the entire people, that’s a very impressive development at the time period.

      Each coddification of Jewish law has increased the democratization of the religion, culminating with the Talmud where the “scholars” took the power away from the “priests,” a HUGE change, because it reduced the hereditary power of the priesthood that had held sway… empirically since Ezra, by custom since at least the first Temple, and possibly earlier.

      I believe that the codifiers of this shift truly believed that they were a continuation of the legal regime dating back to the first leader, Moses, a Prophet/King, and therefore the defacto leader whether the people were led by a Prophet or a King in whatever term used. As Moses held “divine authority,” and his successors did as well, the leaders of this group were careful to trace an unbroken line (as best as they could) to his court, and therefore his authority. Similarly, the Priesthood traced their lineage back to Aaron for authority, and potential Messianic leaders are careful to trace their lineage back to David’s lineage.

      While Judaism in the biblical era was more democratic than the rest of the region, it was still clan-based and lineage based. Talmudic Judaism was MUCH more democratic, and each codification and change has pushed more of the obligations/privileges onto the common man, this is a thread that shows how Judaisms evolution is forward and wonderful, even if imperfect. Look at the modern “universal education,” we try to provide every Jew with more Jewish education than most Rabbis would have had for centuries, that’s pretty cool as part of a universal ethos, even if the economics need some “working out.”

      So does a tradition from Sinai exist? Absolutely, communal authority rested with that line until, after it was broken, it was replaced with Rabbinic Consensus in a further democratization. And that democratic access is built on the legal fiction that this is all until “the Messiah” comes and rebuilds all our lost institutions. Rabbinic objection to building a Temple shows that there is no real desire to reestablish the Priesthood as a competing center of Jewish spirituality, but the legal authority rests on them serving as a kind of regency for the Messiah’s new Court, the new Messiah taking Moses’s authority.

      • AccidentalKorach says:

        Miami Al

        The forerunners to the Tannaim – the Perushim – known to both Philo and Josephus – show no awareness of the chain to Sinai. Similarly no writing of the second Temple period shows any such awareness.

        It goes without saying that the Tanach is silent on this point.

        Yes it is true that the move from Priestly leaders to Scholarly ones is a democratisation of sorts (sorry girls – boys only club – even in 2010 CE). This point however is not relevant to the question of the Sinaitic authorityof the oral law.

      • Dov Kramer says:

        >>It goes without saying that the Tanach is silent on this point (Oral tradition from Sinai)<<

        What do you think the Torah thinks Moshe did for 40 days/nights atop Mt. Sinai?

  24. Accidential Korach says:

    Dov Kramer

    Firstly I am not advocating TMS. I do not believe there was a Sinaitic revelation.

    However let us assume that I did hold that position and then ask “is there a Sinaitic oral law?”

    The answer is maybe. But this “authentic” (and not man-made) revelation is not identifiable.

    The vast majority is man made (or accretion of Hebrew and other cultural values and legal practices etc.).

    [And I say man-made because Bruriah is not mentioned in the Mishnah and even if she was – she is 1 woman versus 100s of men.]

    Mishnah Avot understands that there is a chain of tradition. It needs this to shore up its authority. If you discount the chain – you lose your authority.

    • Dov Kramer says:

      >>The vast majority is man made <<

      Yet authoritative b/c it is based on the system and guidelines received at Sinai (and in the Mishkan).

  25. tayqoo says:

    Dov Kramer says:
    July 16, 2010 at 3:23 am
    >>yes, let’s try to figure out what “god” wants or expects from us (if anything) and how we know that.<<

    Okay. The next step would seem to be trying to figure out what this "god" is all about, so that we can try figuring out what "it" expects/wants of us. Let's go through the possibilities.

    1) The Creator left the scene, and therefore wants/expects nothing of us.

    2) The Creator is still involved with His/Her creation, but is not a rational, logical being, so it would be impossible to figure out what He/She wants (nor would there be a reason to want to fulfill it) unless there was a direct communication telling us this information.

    3) The Creator is still involved, is a rational/logical being, and has a reason/ plan for creation.

    There may be subsets of these three possibilities, but have I left out any major, realistic possibilities
    **************************************
    so far so good. whether "god" is a/the creator is not relevant (yet) to our discussion so why not just refer to "god"? at this point we really know nothing about this entity. you're still in the batter's box.

    • Dov Kramer says:

      It doesn’t really matter much how we refer to the Creator. The name “god” comes with a lot of baggage, so I have no problem avoiding it.

      Which of the three possibilities I mentioned above about the Creator do you think is most likely?

  26. Zak says:

    I couldn’t help but notice that none of you thinks it important to even consider another option: that the impulse to discover something we call “God” is driven more by something inside of us rather than anything out there (The God Gene).

    • tayqoo says:

      yes, that’s a possibility but i would like to be convinced that there is a god that wants me to live a certain way. if i can’t be convinced then i would consider other possibilities including the “god gene”.

  27. tayqoo says:

    Being religious is a choice, and one can choose to be religious even if one thinks it’s unlikely God exists, just as one can choose to invest in a stock one thinks is unlikely to do well.
    ****************
    why would anyone invest in a stock that one thinks is unlikely to do well?

  28. Daniel Schwartz says:

    You remind me of an old joke. Two yeshiva boys, Reuven and Shimon wiled a morning seder away debating the existence of G-d. Nimnu v’gamru that there is no G-d. Upon reaching their conclusion they went to have a drink. Reuven made a bracha and drank. Shimon exclaimed, “But we just decided there is no G-d!!! Why the bracha?” Shimon responded “Takeh there is no G-d. But only a pruste goy would drink without first making a bracha.”

  29. tayqoo says:

    Dov Kramer says:
    July 19, 2010 at 4:39 pm
    It doesn’t really matter much how we refer to the Creator. The name “god” comes with a lot of baggage, so I have no problem avoiding it.

    Which of the three possibilities I mentioned above about the Creator do you think is most likely
    **************
    dov,

    thanks for coming back. i was getting worried.

    like i said, we really know nothing about god so i don’t know whether god is a creator or not and at this point in our discussion it is not relevant. if you have a problem with the term god (“baggage”) let’s choose another term to represent the entity that we are discussing. how about el? i’m understanding el to be that entity that is responsible for what i percieve to be reality.

    you had written, “There is no reason we have to have absolute knowledge of the Creator’s [el’s] existance before committing ourselves to finding out what He/She wants/expects from us. How likely His/Her existance has to be estimated to be before making such a committment varies from person to person (and here the percentages are greatly affected by the perception of what might be required), but to say it’s either yes or no, I don’t buy it.”

    i’m not sure how anyone would go about estimating the reality of el’s existance. what data would you use to calculate the percentages? we can argue this point forever and not get anywhere. the way i see it, either el exists or doesn’t exist. if el doesn’t exist then what are we discussing? so for arguments sake let us say that el exists (my preferance).

    so let’s conttinue this discussion with your choices

    “1) The Creator [el] left the scene, and therefore wants/expects nothing of us.
    2) The Creator [el] is still involved with His/Her creation [reality as we perceive it], but is not a rational, logical being, so it would be impossible to figure out what He/She wants (nor would there be a reason to want to fulfill it) unless there was a direct communication telling us this information.
    3) The Creator [el] is still involved, is a rational/logical being, and has a reason/ plan for creation.
    There may be subsets of these three possibilities, but have I left out any major, realistic possibilities?”

    choice 1 is the equivalent of el’s not existing.
    choice 2 is also the equivalent of el’s not existing unless there was “direct (?) communication telling us” what el wants.

    so in response to your question “which of the three possibilities I mentioned above about the Creator [el] do you think is most likely”, i have no idea how to determine the likelihood of any of them but i would like to explore choice 2 with “direct (?)communication” or choice 3. pick one or the other and let’s see where it leads.

    • Dov Kramer says:

      tayqoo- don’t get “worried.” I am not on the computer all the time. I get notice of additional comments emailed to me, so next time I am “on” I can check them out. (After Tisha b’Av I had 46 notifications; I doubt I’ll get to all of them tonight.)

      I think we are coming from this at different angles. I don’t think it helps to just “pick the option” that enables the conversation to continue. At each step, we must think it through (which masy take a while for each step) before proceeding.

      You are correct that there’s no point in continuing the conversation if “El” doesn’t exist. That doesn’t mean we should therefore just pretend He/She does. It means we should try figuring out whether it is more likely that He/She does, i.e. that there is a Creator, or that He/She doesn’t.

      Just because we can’t “know” doesn’t mean we can’t try determining which possibility makes more sense to us.

      Do you think it’s more likely that the was a “First Cause” or “Prime Mover,” or that things came about on their own?

  30. tayqoo says:

    dov,

    how do i get from point a (choice 2 or 3) to shomer torah u’mitzvot?

    for those that are fasting, have meaningful one.

    • Dov Kramer says:

      >>how do i get from point a (choice 2 or 3) to shomer torah u’mitzvot?<<

      There are a lot of steps in between. Perhaps we can take them one at a time.

  31. tayqoo says:

    Dov Kramer says:
    “tayqoo- don’t get “worried.”
    worried was the wrong word to use. anticipated disappointment would have been closer to the truth. i’ve initiated too many of these online, phone and face to face discussions that went nowhere (except in circles) to be optimistic about it’s outcome.

    “I think we are coming from this at different angles. I don’t think it helps to just “pick the option” that enables the conversation to continue. At each step, we must think it through (which masy take a while for each step) before proceeding.”

    in the interest of saving time, why shouldn’t we discuss those options that enable the conversation to continue, first? i would like very much to ‘believe’ in something. there are many more hypothetical scenarios than the 3 you presented. but concluding that el doesn’t exist isn’t likely to make a ‘believer’ out of me. so lets assume (for discussions sake) that el exists.

    “You are correct that there’s no point in continuing the conversation if “El” doesn’t exist. That doesn’t mean we should therefore just pretend He/She does. It means we should try figuring out whether it is more likely that He/She does, i.e. that there is a Creator, or that He/She doesn’t.”

    again, i’m not sure what you mean by “more likely”. since we have no data to use to determine likelihood, i’m assuming that you mean that given that we really can’t prove, one way or the other, whether el exists we choose the hypothesis that we feel more comfortable with and declare it ‘more likely’. i’m not trying to be argumentative. i’m trying to be honest with myself. given that i don’t know the likelihood and i have no way of calculating the likelihood i’m going to arbitrarily say that the likelihood of el’s existance is 50/50. we haven’t even defined what el is. at this point when i refer to el i am referring to an entity that is ‘responsible’ for reality. since ultimately (for many reasons) i would like to ‘believe’ in some sort of ‘yiddishkeit’ i choose to believe that el exists until that belief becomes untenable. i’m not sure that el ‘wants’ anything and if el does ‘want’ something what does el want and how would i know.

    “Just because we can’t “know” doesn’t mean we can’t try determining which possibility makes more sense to us.
    Do you think it’s more likely that the was a “First Cause” or “Prime Mover,” or that things came about on their own?”

    i’m not sure what a “first cause” or “prime mover” has to do with anything unless you are trying to rule out the possibility that el doesn’t exist and want to define el as the “first cause” or “prime mover”. as far as i’m concerned el doesn’t have to be the first cause or prime mover (though el may be those things too). let’s try to avoid defining el in our image.

    i’d like to examine the el hypothesis, so i will assume/accept/believe that el is real. it’s a long way from that to “yiddishkeit”.

    i’m not sure how many people here are interested in our thoughts on this subject. i’m willing to continue this by email. if you want to do this via email, i can be reached at ayeqa1@yahoo.com. this is a dormant email account so i don’t really check it for mail at all. if you’d prefer to do this by email (to avoid the inevitable trolls) let me know. i will check daily for the next few days (until shabbat). if i don’t get any emails i’ll assume that the discussion is over.
    i’m looking forward to a productive outcome

    p.s. i’m not trying to convince anyone that el exists or not or to convince anyone of anything at all. I WOULD LIKE TO BE CONVINCED. i am not a talmid chacham but i do know my way around. i come from a yeshiva background. all my children and grandchildren are charaidi and are involved in chinuch/kiruv/kollel.

    • Baal Habos says:

      Dov C, if you don’t mind, please include me in your email chat with Taqoo, I can be reached at baalhabos@gmail.com

      • Dov Kramer says:

        BhB,

        We are not communicating via email, only through the comments section of the blog. I subscribed to get all comments sent to my email, so that I don’t have to read all the comments again in order to see if a new one was added.

    • Dov Kramer says:

      Tayqoo-

      Unless you object, I would prefer to continue the conversation here. As you can tell, others are also interested in our conversation.

      If you prefer to only continue via email, I will respect that, but will also ask if it’s okay if others interested in this conversation can be “in” on the conversation, or at least can eavesdrop.

      Let me know. But until then, I will try responding to your many points here.

    • Dov Kramer says:

      “in the interest of saving time, why shouldn’t we discuss those options that enable the conversation to continue, first? i would like very much to ‘believe’ in something.”

      That is very much a different conversation. We can have that one instead, but my brain doesn’t work that way. I can’t pretend to find a rational way to believe s/t if I’m starting from an irrational perspective. If I just take it for granted that “El” exists and that He/She is a logical rational being, and I wanted to “believe,” I could suspend rational thought and just accept whatever it was I wanted to believe. What is holding you back from “believing?”

      “there are many more hypothetical scenarios than the 3 you presented. but concluding that el doesn’t exist isn’t likely to make a ‘believer’ out of me. so lets assume (for discussions sake) that el exists.”

      What other hypothetical scenarios are there?

      DK: “You are correct that there’s no point in continuing the conversation if “El” doesn’t exist. That doesn’t mean we should therefore just pretend He/She does. It means we should try figuring out whether it is more likely that He/She does, i.e. that there is a Creator, or that He/She doesn’t.”

      TQ: “again, i’m not sure what you mean by “more likely”.

      I’ll try to give you an example. I’ve been (happily) married for over 13 years (and no, my wife doesn’t read any blogs, at least not yet), but didn’t get married until I was 32. So I was “on the scene” for a while (actually retiring from dating b/c it was so frustrating), and, as is often the case, was contacted often with shidduchim, and had more names read to me than nights to go out. So which girl should I agree to go out with, if there are several to choose from (having met none of them). Obviously, I would go out with the one I thought was “most likely” to be in-line with my personality and goals. Could I “know” which one was “it?” Of course not (or I wouldn’t waste the time/$$ on dating, even if I enjoyed meeting and learning about new people). I had to use whatever info I had to decide which one to commit to taking out.

      We make such decisions (and commitments) all the time. Not just whom to take out, or even whom to marry, but which job to take, which restaurant to go to, which recipe to try, which school to go to/send our kids, which doctor/lawyer/financial advisor to use, which career path to take, etc. We don’t “know” ahead of time which is best, but we do our due diligence (hopefully), and based on our findings, proceed.

      Choosing a lifestyle is no different. We must do our due diligence, and then proceed after deciding which is the most correct, the most true, or whatever it is that is important to us in how we live. And yes, we use everything we can as a resource in making these decisions, including speaking to friends (or fellow blog readers).

      “we haven’t even defined what el is.”

      Of course not, because part of our due diligince includes figuring that out. we can’t just define something b/c we like that definition. At least not if we want to at least have a better chance of being right.

      “at this point when i refer to el i am referring to an entity that is ‘responsible’ for reality. since ultimately (for many reasons) i would like to ‘believe’ in some sort of ‘yiddishkeit’ i choose to believe that el exists until that belief becomes untenable.”

      Why would you prefer to believe in “El?” (Since your “belief” stems from what you want to believe, why you want to will ultimately shape what you end up believing in.)

      “i’m not sure that el ‘wants’ anything and if el does ‘want’ something what does el want and how would i know.”

      This is part of what we must try to figure out, and the first step is (I would think) trying to figure out characteristics this “El” has.

      “i’m not sure what a “first cause” or “prime mover” has to do with anything unless you are trying to rule out the possibility that el doesn’t exist and want to define el as the “first cause” or “prime mover”. ”

      Why must “El” exist? You gave the Creator the name “El” in our discussion. For me, the whole conversation/thought process starts with whether or not there is a Creator. If there isn’t, why would I think any god exists? (I know the Aish Hatorah reasons why He/She must exist aside from creation, but I don’t think they are strong enough on their own to conclude that there can be a separate entity that wants the role of “el” if it wasn’t the Creator.)

      “i’d like to examine the el hypothesis, so i will assume/accept/believe that el is real. it’s a long way from that to “yiddishkeit”.”
      Yes. But it’s also a long way from there being a Creator to Yiddishkeit. Without understanding why you want to believe in “El,” or what you want this “El” to be, it would be very difficult to connect him/Her with anything.

  32. tayqoo says:

    dov,
    it’s fine by me to continue here as long as i can ignore the trolls. unfortunately there are trolls beshogeg but a troll is a troll is a troll is a troll. if it walks like a troll and talks like a troll i’ll consider it a troll (shogeg/unintentional or maizid/intentional) and will try to ignore of all them until it becomes too much to tolerate. whether our baal achsanya (host) is a fraud or not shouldn’t really matter . i have no reason to doubt what he has told us about himself. i personally would really appreciate more of his input as i think that we are pretty much on the same page. those of you that doubt his integrity and have nothing positive to contribute should get lost (in my opinion).

    i just reread what wrote above and it may sound like i’m full of myself. i’d like to explain….i’m in my 60’s and have led my life believing, more or the less, the ‘party line’. its been only recently (all kids married off) that i have had the luxury of thinking about these things more deeply and clearly and have so far come to the conclusion that i (really all of us) have been sold a bill of goods. it’s taken a while to get most of the anger out of my system (i still have some residual anger which is why i won’t talk to trolls). in any case, according to the actuarial tables i have about 20 years to meet my maker/el. i have no time to waste. i’m not concerned about olam haba (world to come/after death) because i currently don’t believe in one. but i do believe in olam haba if it means the future in this world.

    i’ve never met baalhabos except online (and some brief emails) and i value his participation in our discussion.

    dear rabbi orthoprax, if for some reason you feel that your blog is being hijacked, please accept my apologies and let me know.

  33. tayqoo says:

    when i started writing my most recent comment you hadn’t posted your comment of 623pm yet. i didn’t refer to it because i hadn’t read it yet. hopefully we will continue discussing this back and forth for a while. unless you have a better idea, in the interest of avoiding confusion i’m going to copy and paste anything that you write that i’m responding to in quotes. my comment will start with a t (to indicate tayqoo) followed by a number to indicate which level/sequence in a series of back and forth we are at. i hope that works.

  34. tayqoo says:

    i’m cutting/copying/pasting your comment thay i am responding to. i’m not putting that in quotes. instead i put a d (for dov) and 1 to indicate that we labeling this as the 1st in a series of shakla/vetarya (comment/response). a t or d preceding a paragraph indicate comments made before this entry (the original text). t or d followed by a number is pirush/commentary.
    Dov Kramer says:
    July 21, 2010 at 6:23 pm
    t “in the interest of saving time, why shouldn’t we discuss those options that enable the conversation to continue, first? i would like very much to ‘believe’ in something.”
    d1That is very much a different conversation. We can have that one instead, but my brain doesn’t work that way. I can’t pretend to find a rational way to believe s/t if I’m starting from an irrational perspective. If I just take it for granted that “El” exists and that He/She is a logical rational being, and I wanted to “believe,” I could suspend rational thought and just accept whatever it was I wanted to believe. What is holding you back from “believing?”
    t1i’m not sure what you mean by “very much a different conversation”. i don’t know what you were expecting. in any case why do you think that we are starting from an “irrational perspective”? i hope that we aren’t getting derailed before we start. I KNOW NOTHING ABOUT el. i’ve defined el as the entity that is responsible for everything out there (call it a first cause/prime mover/groundfor existance/etc) but i don’t KNOW el’s character. i don’t KNOW whether el is a “logical rational being” or not. incidently (or not) neither did moshe rabainu according to the torah (however we understand the torah). why do you think that you haven’t already suspended rational thought by just accepting whatever it is that you want to believe? if we were looking at a box and i told you that your life (or the existance of the most important thing to you) depended on your answering the following question CORRECTLY within an hour, how would you go about coming up with the CORRECT answer? let’s raise the stakes. assuming that the most valuable thing to you is sentient, if you answer CORRECTLY this sentient being will enjoy the pleasures of gan eden (learning torah/72 virgins/cheeseburgers, pick your pleasue) for eternity. conversely, if you answer INCORRECTLY, this sentient being will suffer the tortures of gay-hinom (the exact opposite of what goes on in gan eden, pick your torture) for eternity. the question is……is there anything in the box besides air (and whatever is usually in air)? you get only one chance to answer and to top it all off you’ll never know whether your answer is CORRECT or not. how would you go about determining whether there is anything in the box besides air? in that kind of situation i would be up the creek with no paddle. i have no data to work with. i would just flip a coin. is that suspending rational thought? i don’t think so. if i heard a ticking sound coming from the box and i answered “no” anyway, that would be suspending rational judgement. i understand ‘suspending ratinal thought” as ignoring the data but if there is no data how am i suspending rational thought? by flipping a coin i am essentially believing (really hoping, which is what i think belief is) that my coin toss will produce the CORRECT , heads yes tails no. i’m not saying that our discussion of yiddishkeit is exactly analogous to the above situation but it’s close. i just don’t understand what you mean by suspending rational judgement. you ask “what is holding you (tayqoo) back from believing”. can you be more specific? believing in what exactly (aliens, the loch ness monster, the emporer’s new clothes, torah min hashamayim, pe ha’aton, moshiach, techiyat hametim, the trinity, yechi adonaynu, that women can’t sit on a bet din because she is a women, yadayadaya).
    t “there are many more hypothetical scenarios than the 3 you presented. but concluding that el doesn’t exist isn’t likely to make a ‘believer’ out of me. so lets assume (for discussions sake) that el exists.”
    d1What other hypothetical scenarios are there?
    t1multiple els and if i gave it some thought i probably could come up with others.
    d1DK: “You are correct that there’s no point in continuing the conversation if “El” doesn’t exist. That doesn’t mean we should therefore just pretend He/She does. It means we should try figuring out whether it is more likely that He/She does, i.e. that there is a Creator, or that He/She doesn’t.”
    t1pretend sounds negative, like make believe but that is basically what we do when we propose a hypothesis. let’s pretend/make believe/hypothesize/let’s say/if-then that x=2. let’s see where that takes us. 2x=4. if 2x=5 then we can’t pretend/make believe/hypothesize/lets sayif-then that x=2. in any case, for whatever reason, each of us individually wants to “pretend” that el more likely than not exists. you because you think there is data to work so that you can make an educated guess and me since i have no data and it’s 50/50 ( i don’t have enough data to honestly determine the likelihood one way or another) am going to “pretend” that el more likely than not exists because i’d like to rule that possibility out before i examine the alternative. we have to start somewhere so why not “pretend” el exists you believe that you have data supporting the likelihood that ytour belief is true and i because i want to believe (without twisting myself into too much of a pretzel-COGNITIVE DISSONANCE).
    t TQ: “again, i’m not sure what you mean by “more likely”.
    d1I’ll try to give you an example. I’ve been (happily) married for over 13 years (and no, my wife doesn’t read any blogs, at least not yet), but didn’t get married until I was 32. So I was “on the scene” for a while (actually retiring from dating b/c it was so frustrating), and, as is often the case, was contacted often with shidduchim, and had more names read to me than nights to go out. So which girl should I agree to go out with, if there are several to choose from (having met none of them). Obviously, I would go out with the one I thought was “most likely” to be in-line with my personality and goals. Could I “know” which one was “it?” Of course not (or I wouldn’t waste the time/$$ on dating, even if I enjoyed meeting and learning about new people). I had to use whatever info I had to decide which one to commit to taking out.
    d1We make such decisions (and commitments) all the time. Not just whom to take out, or even whom to marry, but which job to take, which restaurant to go to, which recipe to try, which school to go to/send our kids, which doctor/lawyer/financial advisor to use, which career path to take, etc. We don’t “know” ahead of time which is best, but we do our due diligence (hopefully), and based on our findings, proceed.
    d1Choosing a lifestyle is no different. We must do our due diligence, and then proceed after deciding which is the most correct, the most true, or whatever it is that is important to us in how we live. And yes, we use everything we can as a resource in making these decisions, including speaking to friends (or fellow blog readers).
    t1you’re absolutely right. we must do our due diligence. we collect the data and come to a conclusion based on the data. BUT WE NEED DATA. we can’t make it up or imagine it. even when we have data it has to be INTERPRETED correctly.
    t “we haven’t even defined what el is.”
    d1Of course not, because part of our due diligince includes figuring that out. we can’t just define something b/c we like that definition. At least not if we want to at least have a better chance of being right.
    ti. in everyday life how do you go about figuring things out? you first have to define what it is you’re talking about. the more complicated a definition is the greater the possibility of misunderstanding. for the purpose of our discussion i understand el to be the entity responsible for existance. el may be much more than that but you and i can “pretend” that to be true. if you’d like to add to the definition then you’ll have to convince me to agree to “pretend” like you
    t “at this point when i refer to el i am referring to an entity that is ‘responsible’ for reality. since ultimately (for many reasons) i would like to ‘believe’ in some sort of ‘yiddishkeit’ i choose to believe that el exists until that belief becomes untenable.”
    d1 Why would you prefer to believe in “El?” (Since your “belief” stems from what you want to believe, why you want to will ultimately shape what you end up believing in.)
    t1 because i don’t want to believe (personal preference) that there is no possibility that the life i’ve lead until now was meaningless, as long as the cognitive dissonance is bearable.
    t “i’m not sure that el ‘wants’ anything and if el does ‘want’ something what does el want and how would i know.”
    d1 This is part of what we must try to figure out, and the first step is (I would think) trying to figure out characteristics this “El” has.
    t1 step up to the plate and hit that ball out of the parl. let’s go!
    t “i’m not sure what a “first cause” or “prime mover” has to do with anything unless you are trying to rule out the possibility that el doesn’t exist and want to define el as the “first cause” or “prime mover”. ”
    di Why must “El” exist? You gave the Creator the name “El” in our discussion. For me, the whole conversation/thought process starts with whether or not there is a Creator. If there isn’t, why would I think any god exists? (I know the Aish Hatorah reasons why He/She must exist aside from creation, but I don’t think they are strong enough on their own to conclude that there can be a separate entity that wants the role of “el” if it wasn’t the Creator.)
    t1 i never said el must exist. i’m pretending. i never refered to el as the creator. whether el is the creator or assigned some agent to create or even if there was no creation doesn’t really change anything. does it? let me understand this. are you saying that you believe in el only because of “creation”. if there were no creation then you would be an atheist?
    t “i’d like to examine the el hypothesis, so i will assume/accept/believe that el is real. it’s a long way from that to “yiddishkeit”.”

    d1Yes. But it’s also a long way from there being a Creator to Yiddishkeit. Without understanding why you want to believe in “El,” or what you want this “El” to be, it would be very difficult to connect him/Her with anything.
    t1 i’m not saying that el isn’t the creator. el is el even if there was no creation. it could be that we are saying the same thing in a sense. i’m saying that el is responsible for existence and your saying that he created existence. where do we go from here?

  35. Dov Kramer says:

    Wow. I’ll give this a try (although I may mess up your coding system, my apologies if that happens).

    t1- why do you think that you haven’t already suspended rational thought by just accepting whatever it is that you want to believe?

    Because I haven’t just “accepted” it. I’ve thought it through. Over years. And thought it through again and again. This conversation has caused me to think it through yet again. I haven’t come to the conclusions I’ve reached because I’ve wanted to. I’ve tried to leave the conclusions open ended and let them be whatever sense of reasoning I have takes me. At this point, the only real doubts I have are based on seeing that most people don’t (or can’t) think straight, so why should I trust that my reasoning is any more valid. Conversations such as these help me test whether my sense of reason is sound or faulty. But, based on the sense of reason I currently have, all the “big picture” (and almost all of the “small picture”) questions/issues are answered/explained by the traditional Jewish belief system. (Granted that what I consider “big” or “small” picture may be seen by others as the reverse, which is all part of diffrent people thinking differently.)

    t1 (con’t)- if we were looking at a box and i told you that your life (or the existance of the most important thing to you) depended on your answering the following question CORRECTLY within an hour, how would you go about coming up with the CORRECT answer?

    But that’s not really the case. We are given some clues, and we have our ability to reason to figure out what those clues mean. And you are putting all belief systems into the same single question that must be answered correctly, when every belief system should be judged on its own merits independantly. It’s not just figuring out if there’s a Creator, it’s figuring out if the Creator wants something from us, what that something is, and if anyone else has come to the same (or similar) conclusions.

    t1 (con’t2)- i understand ‘suspending ratinal thought” as ignoring the data but if there is no data how am i suspending rational thought?

    But there is data. Our very existance is data regarding whether or not there is a Creator. It may be false data, but it’s still data that must be considered. The nature of our existance is very valuable data, because it tells us much about the Creator (if there is one). Some may consider the various religious claims to be data, but most of that is false data, and we’d have to figure which of it (if any) is data worth taking into consideration. Since there is plenty of data to study, investigate, and ponder, we must use our (hopefully rational) sense of judgement to try interpreting that data and see which conclusions it leads us to.

    t1 (con’t3)- by flipping a coin i am essentially believing (really hoping, which is what i think belief is) that my coin toss will produce the CORRECT , heads yes tails no.

    That’s why I try avoiding the word “belief,” and prefer the word “conclusion.” In a strict sense of the word, though, I understand “belief” not as the “hope of being correct,” but as the leap of faith that bridges the gap between thinking something is likely to be true and knowing 100% that it is.

    t1 (con’t4)- you ask “what is holding you (tayqoo) back from believing”. can you be more specific? believing in what exactly (aliens, the loch ness monster, the emporer’s new clothes, torah min hashamayim, pe ha’aton, moshiach, techiyat hametim, the trinity, yechi adonaynu, that women can’t sit on a bet din because she is a women, yadayadaya).

    Whatever it is you said you “wanted to believe.” You are willing to forego discussion about whether it makes more sense that there is Creator than that there isn’t, and about what this Creator is all about, all because you “want to believe.” So I asked why those can be accepted as “givens” without due process while the other steps necessary
    to believe cannot just be accepted as “givens.”

    d1What other hypothetical scenarios are there?
    t1multiple els and if i gave it some thought i probably could come up with others.

    That might be a theoretical possibility regarding what exists, but I don’t consider it a separate possibility regarding the First Cause (rather, it is a subset of having an irrational Creator, since multiple creators, each creating a different aspect of creation, would lead to each wanting something different, and the impossibility to do what every one of them wants).

    t1- for whatever reason, each of us individually wants to “pretend” that el more likely than not exists. you because you think there is data to work so that you can make an educated guess

    But if I think I have data, is that really pretending? I may be mistaken, but using poor data, or using data poorly is a mistake/error, not a game of make-believe, no?

    t1 (con’t)- we have to start somewhere so why not “pretend” el exists

    Okay, I’ll try. From now on I will pretend that (for our conversation) it is an automatic “given” that the Creator exists (baruch hu).

    t1- for the purpose of our discussion i understand el to be the entity responsible for existance.

    Okay.

    d1 Why would you prefer to believe in “El?”
    t1 because i don’t want to believe (personal preference) that there is no possibility that the life i’ve lead until now was meaningless, as long as the cognitive dissonance is bearable.

    If we are going to set “givens” without having to fully determine that they are likely true, and the reason you want to do so is to give your life meaning, may I suggest another “given?” That the Creator (“El”) created the world for a reason, with a purpose. (As you’ve probably realized, this “given” was one of the “steps” along the way in my “thought process,” coming shortly after “concluding” that the Creator, er, I mean “El,” is a rational, logical being.)

    Is that an okay “given” to also work with, for our conversation?

    t1- let me understand this. are you saying that you believe in el only because of “creation”. if there were no creation then you would be an atheist?

    Hard to say for sure. There are multiple factors working together, each feeding off each other. Our existance being much more easily (and perhaps only) explained if there is/was a Creator is one of those factors.

    t1 i’m not saying that el isn’t the creator. el is el even if there was no creation.

    If there was no creation, why should I care so much about pleasing another being?

    t (con’t)- it could be that we are saying the same thing in a sense. i’m saying that el is responsible for existence and your saying that he created existence. where do we go from here?

    For me, the next step is trying to understand El, but that is only possible (as you point out) if there is data. If El did not create the world (or is not responsible for its existance), then we have no data, unless there is a communication from Him/Her. If however, El is responsible for existance, then we can mine plenty of data from that existance to try to understand El.

    And, BTW, there is no “getting up to the plate and hitting one out of the park” here. Few teams win by relying on just the longball. Aside from good pitching (i.e. asking the right questions), it’s getting the little things right (the fundamentals of the game) and timely hitting. I’d like to think we’re playing on the same team.

  36. tayqoo says:

    dear dov,
    i’m not sure whether you are trying to convince yourself or convince me. it seems that we are getting bogged down in semantics so lets start over. we can keep these comments much shorter so that we don’t come to too many misunderstandings.

    we can start with a blank slate. we can come to the same conclusion for different reasons. you can conclude that el exists because based on your interpretation of the data el must exist. based on my interpretation of the data el may exist or el may not exist but i’d still prefer to conclude that el exists because i don’t want to believe (personal preference) that there is no possibility that the life i’ve lead until now was meaningless, as long as the cognitive dissonance is bearable. that’s the whole purpose in this discussion with you. i’d like to regain my belief because right now i feel that from a certain perspective i may have wasted my life so far pursuing the (non existant) holy grail.

    so, even though i’m not as convinced as you and certainly not for the same reasons i will conclude that el exists and that el is the creator and that there is only one el. now what?
    in our past comments we were going off into too many different directions (maybe my fault). lets keep our comments short and simple. let’s not get ahead of ourselves. let’s try not to get caught up in semantics.

  37. tayqoo says:

    i’m really sorry dov. i thought that i could learn something from. now i’m wondering whatever made me think that. thanks for your efforts but there is really no point in continuing this charade. you’ve only managed to convince me again that it’s a mistake to expect an honest discussion with someone whose mind is made up that they MUST be right. i’m really sorry for wasting your time and sorrier for wasting mine.

    • Dov Kramer says:

      I’m sorry that you feel that way. I was trying to cut to the chase, but I must have misunderstood what you were looking for.

      Do you want to try again?

  38. tayqoo says:

    “learn something from you”.

  39. tayqoo says:

    avot 1:11
    avot 2:5

Leave a reply to Ephraim Cancel reply